微积分 A(2) 学习笔记

多元函数的极限与连续性

多元函数的定义

多元函数是依赖有限多个(多余一个)连续变量的函数

\[f(x^1,x^2,\cdots,x^n) \]

\[f:U\subset\mathbb{R}^n\to\mathbb{R} \]

在多元微积分中常常用顶标表示下标。

空间中的距离

集合 \(M\) 上的距离是一个映射 \(d:M\times M\to R\) 满足:

  1. 正定性:\(\forall \bold{x},\bold{y}\in M,d(\bold{x},\bold{y})\ge 0\)\(d(\bold{x},\bold{y})=0\) 当且仅当 \(\bold{x}=\bold{y}\)
  2. 正齐次性:\(\forall \bold{x},\bold{y}\in M,d(\bold{x},\bold{y})=d(\bold{y},\bold{x})\)
  3. 三角形不等式:\(\forall \bold x,\bold y,\bold z\in M\)\(d(\bold{x},\bold{z})\le d(\bold{x},\bold{y})+d(\bold{y},\bold z)\)

其中 \(M=\mathbb{R}^n\) 或者 \(\mathbb{R}^n\) 中的曲线、曲面或者函数空间。

范数

范数是一个 \(\mathbb{R}^n\to\mathbb{R}\) 的映射,记作 \(\lVert\bold{x}\rVert\),满足:

  1. \(\forall\bold{x}\in\mathbb{R}^n\)\(\lVert\bold{x}\rVert\ge 0\),且 \(\lVert\bold{x}\rVert=0\) 当且仅当 \(\bold{x}=0\)
  2. \(\forall\lVert\bold{x}\rVert\in\mathbb{R}^n,\forall\lambda\in\mathbb{R}\)\(\lVert\lambda\bold{x}\rVert=|\lambda|\lVert x\rVert\)
  3. \(\forall\bold{x},\bold{y}\in\mathbb{R}^n,\lVert\bold{x}+\bold{y}\rVert\le\lVert\bold{x}\rVert+\lVert\bold{y}\rVert\)

根据范数,我们可以确定一个满足平移不变性质的距离:

\[d(\bold{x},\bold{y})=\lVert\bold{x}-\bold{y}\rVert \]

事实上,范数的选择不是唯一的,考虑

\[\lVert\bold x\rVert_p=(|x^1|^p+|x^2|^p+\cdots+|x^n|^p)^{\frac{1}{p}} \]

对于所有 \(p\ge 1\)\(\lVert\bold x\rVert_p\) 均是 \(\mathbb{R}^n\) 上的范数(即满足范数的三条限制)。特别地,\(1\)-范数为向量每一维绝对值相加后的结果,\(2\)-范数为常义上的欧几里得距离,\(\infty\)-范数为向量每一维绝对值的最大值。

内积

内积是一个 \(\mathbb{R}^n\times\mathbb{R}^n\to\mathbb{R}\) 的映射 \(B\),满足:

  1. 对称性:\(B(\bold x,\bold y)=B(\bold y,\bold x)\)
  2. 双线性:\(B(\lambda\bold{x}+\mu\bold{y},\bold{z})=\lambda B(\bold{x},\bold{z})+\mu B(\bold{y},\bold{z}),B(\bold{z},\lambda\bold{x}+\mu\bold{y})=\lambda B(\bold{z},\bold{x})+\mu B(\bold{z},\bold{y})\)
  3. 正定性:\(B(\bold{x},\bold{x})\ge 0\),且 \(B(\bold{x},\bold{x})=0\) 当且仅当 \(\bold{x}=0\)

\(B(\bold{x},\bold{y})\) 记作 \(\bold{x}·\bold{y}\) 或者 \(\lang\bold{x},\bold{y}\rang\)

由内积可以得到欧几里得范数:\(\lVert\bold{x}\rVert=\sqrt{\bold{x}·\bold{x}}\)。在所有 \(p\)-范数中只有 \(2\)-范数能通过这种方式得到。

与极限有关的定义

对于有限维空间 \(V=\mathbb{R}^n\),定义

  • 邻域、内点:称 \(U\)\(\bold{x_0}\in V\) 的一个邻域或者 \(\bold{x_0}\)\(U\) 的一个内点当且仅当存在 \(r>0\) 使得对于所有 \(\lVert\bold{x}-\bold{x_0}\rVert<r\),都有 \(\bold{x}\in U\)

  • 开集:称 \(U\) 是一个开集,当且仅当 \(U\) 中每个点都是内点。

  • 聚点:称 \(\bold{x_0}\)\(U\) 的一个内点,当且仅当 \(\forall\epsilon>0\),都 \(\exists\bold{x}\in U\) 使得 \(0<\lVert\bold{x}-\bold{x_0}\rVert<\epsilon\)

  • 闭集:称 \(U\) 是一个闭集,当且仅当 \(U\) 的所有聚点都在 \(U\) 内部。

对于两个空间 \(U,V\) 和一将 \(U\) 映射到 \(V\) 的函数 \(f\),设 \(\lVert·\rVert_U\)\(\lVert·\rVert_V\) 为两个空间上的范数,那么

  • \(\lim\limits_{\bold{x}\to\bold{x_0}}f(\bold{x})=\bold{y}\),当且仅当 \(\forall\epsilon>0\),都 \(\exists\delta>0\) 使得 \(0<\forall\lVert\bold{x}-\bold{x_0}\rVert_U<\delta\),都有 \(\lVert\bold{y}-\bold{f(x)}\rVert_V<\epsilon\)
  • \(f(\bold{x})\)\(\bold{x_0}\) 处连续,当且仅当 \(\forall\epsilon>0\),都 \(\exists\delta>0\) 使得 \(0<\forall\lVert\bold{x}-\bold{x_0}\rVert_U<\delta\),都有 \(\lVert\bold{f(x_0)}-\bold{f(x)}\rVert_V<\epsilon\)

对于有限维空间 \(V=\mathbb{R}^n\) 中的点列 \(\bold{x}_n\) 定义

  • \(\lim\limits_{n\to+\infty}\bold{x}_n=\bold{x}\),当且仅当 \(\forall\epsilon>0\),都 \(\exists N\ge 1\) 满足 \(\forall n\ge N\),都有 \(\lVert \bold{x}_n-\bold{x}\rVert<\epsilon\)
  • \(\{\bold{x}_n\}\) 是一个柯西序列,当且仅当 \(\forall\epsilon>0\),都 \(\exists N\ge 1\) 满足 \(\forall i\ge j\ge N\),都有 \(\lVert \bold{x}_i-\bold{x}_j\rVert<\epsilon\)

这些都与一元微积分的情况大同小异,只不过把绝对值改成了范数而已。

函数连续性的定义

\(f:(A\subseteq V)\to W\),对于 \(\bold{x_0}\in V\),称 \(\lim\limits_{\bold{x}\to\bold{x_0}}f(\bold{x})=\bold{L}\),当且仅当:

  • \(\bold{x_0}\)\(f\) 定义域 \(A\) 的聚点。
  • \(\forall\epsilon>0\),存在 \(\delta>0\) 使得 \(\forall\bold{x}\in A\),若 \(0<\lVert\bold{x}-\bold{x_0}\rVert<\delta\),则 \(\lVert f(\bold{x})-\bold{L}\rVert<\epsilon\)

\(f\)\(\bold{x_0}\) 处连续,当且仅当 \(\bold{x_0}\in A\),并且 \(\forall\epsilon>0\),存在 \(\delta>0\) 使得 \(\forall\bold{x}\in A\),若 \(0<\lVert\bold{x}-\bold{x_0}\rVert<\delta\),则 \(\lVert f(\bold{x})-f(\bold{x_0})\rVert<\epsilon\)初等函数的四则运算与复合在定义域内都是连续函数,使用这一点证明连续性比直接用定义方便。

任何范数都与无穷范数等价

对任意范数 \(\lVert ·\rVert\),都存在常数 \(M_1,M_2\) 满足

\[M_1\lVert\bold{x}\rVert_{\infty}\le\lVert\bold{x}\rVert\le M_2\lVert{x}\rVert_{\infty} \]

换句话说,多元函数的极限与连续性与范数的选择无关。

一些与连通有关的定义

道路连通集:称一个集合 \(U\) 是道路连通的,当且仅当 \(\forall P,Q\in A\),存在连续映射 \(\phi:[0,1]\to U\) 使得 \(\phi(0)=P,\phi(1)=Q\)

有以下两条性质:

  1. 连续函数把道路连通集映到道路连通集。
  2. 连续函数把有界闭集映到有界闭集。

重极限与累次极限

对于多元函数而言,称将所有变量同时取极限所得的极限称为重极限,称先对其中一个变量取极限,再对其他变量取极限得到的极限称为累次极限

以二元函数 \(f(x,y)\) 为例,重极限指

\[\lim\limits_{(x,y)\to(x_0,y_0)}f(x,y) \]

累次极限指

\[\lim\limits_{x\to x_0}\lim\limits_{y\to y_0}f(x,y) \]

\[\lim\limits_{y\to y_0}\lim\limits_{x\to x_0}f(x,y) \]

对于二元函数而言,有以下结论:

  • 如果三个极限均存在,那么三个极限必须相等。
  • 如果两个累次极限均存在但不相等,那么重极限必然不存在。

但是如果两个累次极限均存在并且相等,并不能推出重极限一定存在(反例:考虑函数 \(f(x,y)\),满足当 \(y=x^2\)\(x\ne 0\)\(f(x,y)=1\),其余位置均为 \(0\),那么 \(f(x,y)\) 两个累次极限均为 \(0\),但是重极限不存在)。

求多元函数在某点处的极限,一种常见的方法是放缩然后用夹逼定理转化为一元的情况处理

压缩不动点定理

对于空间 \(V\) 中的非空闭集 \(A\),映射 \(f:A\to V\) 满足:

  1. \(f(A)\subset A\)
  2. \(\exists\lambda\in(0,1)\) 满足 \(\forall\bold{x},\bold{y}\in A\)\(\lVert f(\bold{x})-f(\bold{y})\rVert\le\lambda\lVert\bold{x}-\bold{y}\rVert\)

那么存在唯一的 \(\bold{x^*}\in A\) 满足 \(f(\bold{x^*})=\bold{x^*}\),并且 \(\lVert f^n(\bold{x})-\bold{x^*}\rVert\le\dfrac{\lambda^n}{1-\lambda}\lVert f(\bold{x})-\bold{x}\rVert\)

多元函数的微分

微分

对于线性空间 \(V\) 中的开集 \(A\),称映射 \(f:A\to W\)\(\bold{a}\in A\) 处可微,当且仅当存在线性映射 \(L:V\to W\) 满足当 \(\bold{x}\) 趋近于 \(0\) 时:

\[f(\bold{a}+\bold{x})=f(\bold{a})+L(\bold{x})+o(\bold{x}) \]

\(L\)\(\bold{a}\) 处的微分,记作 \(\mathrm Df(\bold{a})=L\)

多元函数的微分也满足链索法则:设 \(A,B\) 为线性空间 \(V,W\) 中的开集,\(f:A\to W\)\(\bold{a}\in A\) 处可微,\(g:B\to Z\)\(\bold{b}=f(\bold{a})\in B\) 处可微,那么 \(g\circ f\)\(\bold{a}\) 处可微,并且

\[\mathrm D(g\circ f)(\bold{a})=\mathrm Dg(\bold{b})\circ \mathrm Df(\bold{a}) \]

导数

对于映射 \(f:\mathbb{R}\to W\)\(a\in\mathbb{R}\),如果极限

\[\lim\limits_{t\to 0}\dfrac{f(a+t)-f(a)}{t} \]

存在,那么称函数在 \(a\) 处可导,并且称该极限的值为函数在 \(a\) 处的导数。

梯度

对于内积空间 \(V\)\(V\) 上的所有线性函数 \(L:V\to R\) 都可以表示为内积的形式,即存在向量 \(\bold{b}\in V\),使得 \(\forall\bold{x}\in V\),都有

\[L(\bold{x})=\lang\bold{b},\bold{x}\rang \]

符合要求的 \(\bold{b}\) 称为 \(L\) 的梯度。

对可微函数 \(f:V\to\mathbb{R}\),微分 \(\mathrm df(\bold{a})\) 的梯度称为 \(f\)\(\bold{a}\) 处的梯度,记作 \(\text{grad} f(\bold{a})\)\(\nabla f(\bold{a})\)

\[\mathrm df(\bold{a})(\bold{x})=\lang\text{grad} f(\bold{a}),\bold{x}\rang \]

方向导数

对于映射 \(f:A\to W\),其中 \(A\) 为非空空间 \(V\) 中的非空开集,\(\bold{a}\in A,\bold{v}\in V\),如果极限

\[\lim\limits_{t\to 0}\dfrac{f(\bold{a}+t\bold{v})-f(\bold{a})}{t} \]

存在,则称该极限为 \(\bold{a}\) 处沿向量 \(\bold{v}\) 的导数。记作 \(\partial_{\bold{v}}f(\bold{a})\)

\(\lVert\bold{v}\lVert=1\),则称 \(\partial_{\bold{v}}f(\bold{a})\)\(\bold{a}\) 处沿方向 \(\bold{v}\) 的方向导数。

\(f\)\(\bold{a}\) 处可微时,\(f\) 沿每个方向都有方向导数,且 \(\partial_{\bold{v}}f(\bold{a})\) 关于 \(\bold{a}\) 是线性的。

但是如果 \(\partial_{\bold{v}}f(\bold{a})\) 都存在,并且关于 \(\bold{a}\) 是线性,并不能说明 \(f\)\(\bold{a}\) 处可微。

  • 反例 \(1\)\(f(x,y)=\begin{cases}\dfrac{x^3+y^3}{x^2+y^2}&(x^2+y^2\ne 0)\\0&(x^2+y^2=0)\end{cases}\),则 \(\partial_{(x,y)}f(0,0)=\dfrac{x^3+y^3}{x^2+y^2}\),这样 \(\partial_{(0,1)}f(0,0)=\partial_{(1,0)}f(0,0)=\partial_{(1,1)}f(0,0)=1\),关于 \((x,y)\) 并不是线性的,所以即使每个方向导数都存在,也不一定存在重极限。
  • 反例 \(2\):考虑一个函数,\(f(0,0)=1\),对于过原点的倾角为 \(\theta\) 的直线,我们让这条直线上离原点距离 \(\le g(\theta)\) 的点的 \(f\) 值为 \(1\),其余点的 \(f\) 值为 \(0\)。因为方向有无限多个,所以我们可以构造函数 \(g\) 使得 \(\lim\limits_{\theta\to 0}g(\theta)=0\),但 \(g(0)\ne 0\),这样每个方向上的方向导数都是 \(0\),但函数 \(f\)\((0,0)\) 处甚至不是连续的!更不用说重极限了。

偏导数

在多元映射 \(f:\mathbb{R}^m\to\mathbb{R}\) 中,只让一个坐标 \(x^k\) 变化,其余坐标不变,则可以让 \(f\) 成为关于 \(x^k\) 的一元函数。如果极限

\[\lim\limits_{t\to 0}\dfrac{f(x^1,x^2,\cdots,x^k+t,\cdots,x^m)-f(x^1,x^2,\cdots,x^k,\cdots,x^m)}{t} \]

存在,则称该极限为 \((x^1,x^2,\cdots,x^m)\) 处关于 \(x^k\) 的偏导数,记作 \(f_{x^k}(\bold{x})\)。也记作 \(\dfrac{\partial f}{\partial x^k}(\bold{x})\)。对 \(x^k\) 求偏导相当于是将 \(x^k\) 看作变量,其余坐标看作参数后对一元函数求导。

偏导数与微分的关系

对于 \(m\) 维线性空间 \(V\) 中的一组基 \(\bold{e_1},\bold{e_2},\cdots,\bold{e_m}\),并且相应的坐标系满足 \(\bold{x}=x^1\bold{e_1}+x^2\bold{e_2}+\cdots+x^m\bold{e_m}\)。如果映射 \(f:V\to\mathbb{R}\) 可微,那么 \(\forall\bold{v}=\xi^1\bold{e_1}+\xi^2\bold{e_2}+\cdots+\xi^m\bold{e_m}\),有

\[\mathrm d f(\bold{x})(\bold{v})=\mathrm df(\bold x)(\xi^1\bold e_1+\xi^2\bold e_2+\dots+\xi^m\bold e_m)=\sum\limits_{i=1}^m\xi^if_{x^i}(\bold{x}) \]

上述式子对于任意一组基 \(\{\bold{e_m}\}\) 都成立,即使 \(\{\bold{e_m}\}\) 不是单位正交的。

偏导数与梯度的关系

对于向量 \(\bold{x}\in\mathbb{R}^m\) 以及 \(\bold{v}=\xi^1\bold{e_1}+\xi^2\bold{e_2}+\cdots+\xi^m\bold{e_m}\)。因为

\[\lang\text{grad}f(\bold{x}),\bold{v}\rang=\mathrm df(\bold{x})(\bold{v})=\sum\limits_{i=1}^m\xi^if_{x^i}(\bold{x}) \]

所以对于笛卡尔坐标系而言(\(\bold{e_i}\) 只有第 \(i\) 维为 \(1\),其余维均为 \(0\)),梯度 \(\text{grad}f(x)\) 可以表示为列向量

\[\begin{pmatrix}f_{x^1}(\bold{x})\\\vdots\\f_{x^m}(\bold{x})\end{pmatrix} \]

对于任意一组基 \(\{\bold{e_m}\}\),设 \(\text{grad}f(x)=c^1\bold{e_1}+c^2\bold{e_2}+\cdots+c^m\bold{e_m}\),那么 \(\lang\text{grad}f(x),\bold{v}\rang=\sum\limits_{i=1}^mc^i\bold{e_i}\bold{v}=\sum\limits_{i=1}^m\sum\limits_{j=1}^mc^i\xi^j\bold{e_i}\bold{e_j}\),又因为 \(\lang\text{grad}f(\bold{x}),\bold{v}\rang=\sum\limits_{i=1}^m\xi^if_{x^i}(\bold{x})\),所以 \(f_{x^j}(\bold{x})=\sum\limits_{i=1}^mc^i\bold{e_ie_j}\),设矩阵 \(G\) 满足 \(G_{i,j}=\bold{e_j}\bold{e_j}\),那么 \((G\bold{c})_j=f_{x^j}(\bold{x})\),这样梯度 \(\text{grad}f(x)\) 可以表示为向量 \(G^{-1}\begin{pmatrix}f_{x^1}(\bold{x})\\\vdots\\f_{x^m}(\bold{x})\end{pmatrix}\)

一阶微分的形式不变性

在任何坐标系 \((x^1,x^2,\cdots,x^m)\) 下,对任何可微函数都有

\[\mathrm df=\sum\limits_{k=1}^mf_{x^k}\mathrm dx^k \]

\(\mathrm df\) 为全微分,\(\sum\limits_{k=1}^mf_{x^k}\mathrm dx^k\) 为偏微分。

该性质表明,一阶微分是与坐标系选取无关的几何概念。

Jacobi 矩阵

对于线性空间 \(V,W\)\(A\subset V\),映射 \(f:A\to W\) 可微。设 \(V\) 中存在坐标系 \((x^1,x^2,\cdots,x^m)\)\(W\) 中存在坐标系 \((y^1,y^2,\cdots,y^n)\),那么 \(\bold{y}=f(\bold{x})\) 可以表示成如下的一组函数:

\[\begin{cases} y^1=f^1(x^1,x^2,\cdots,x^m)\\ y^2=f^2(x^1,x^2,\cdots,x^m)\\ \vdots\\ y^n=f^n(x^1,x^2,\cdots,x^m) \end{cases} \]

这样 \(f\) 的微分可以表示为矩阵

\[Jf(x)=\begin{pmatrix} \dfrac{\partial y_1}{\partial x_1}&\dfrac{\partial y_1}{\partial x_2}&\cdots&\dfrac{\partial y_1}{\partial x_m}\\ \dfrac{\partial y_2}{\partial x_1}&\dfrac{\partial y_2}{\partial x_2}&\cdots&\dfrac{\partial y_2}{\partial x_m}\\ \vdots&\vdots&\ddots&\vdots\\ \dfrac{\partial y_n}{\partial x_1}&\dfrac{\partial y_n}{\partial x_2}&\cdots&\dfrac{\partial y_n}{\partial x_m}\\ \end{pmatrix} \]

复合映射的 Jacobi 矩阵是每步映射的 Jacobi 矩阵的乘积,即 \(J(g\circ f)(\bold x)=Jg(f(\bold(x)))J(\bold{x})\)

偏导数与可微性

偏导数是一个很弱的概念。即便所有该点方向导数都存在,也不能推出该点可微。那么添加什么条件能够得出该点可微呢?

以二元函数为例,对于函数 \(f\) 定义域中的某点 \((a,b)\) 及趋近于 \(0\)\((x,y)\)

\[\begin{aligned} &f(a+x,b+y)-f(a,b)\\ =&f(a+x,b+y)-f(a+x,b)+f(a+x,b)-f(a,b)\\ =&f_2(a+x,b+\eta)y+f_1(a,b)+o(x) \end{aligned} \]

如果我们希望 \(f_2(a+x,b+\eta)=f_2(a,b)y+o(y)\),那么需要满足 \(f_2(a,b)\)\((a,b)\) 中某个邻域中存在并且连续。将这个结论推广到多维的情况可知,如果偏导数在某一维中存在,其余维中都在该点的邻域中连续,就能推出函数在该点中可微。为了对称,一般来说使用的是所有维都在邻域中连续。

定义函数在一个点处连续当且仅当在其它变量固定,一个变量可变时,函数连续。类似地可以得出,若函数在某点处关于一个变量偏连续,关于其他变量在邻域中存在有界偏导,则函数在该点处可微。

高阶偏导数

高阶偏导数的定义:偏导数的偏导数。

\[f_{k_1,k_2,\cdots,k_r}=\dfrac{\partial^rf}{\partial x^{k_r}\partial x^{k_{r-1}}\cdots\partial x^{k_1}}=\dfrac{\partial}{\partial x^{k_r}}\dfrac{\partial}{\partial x^{k_{r-1}}}\cdots\dfrac{\partial}{\partial x^{k_1}}f \]

即按照从右往左的顺序执行偏导。

Clairaut 定理:对于一组 \(k_1,\cdots,k_r\),若对任意排列 \(σ\)\(f_{k_{\sigma_1},k_{\sigma_2},\cdots,k_{\sigma_r}}\) 都连续,则它们的值都相等。(此时,求导与顺序无关)。

对于函数 \(f\) 而言,

  • 如果 \(f\) 在定义域中每个点处都存在一阶偏导数,并且所有一阶偏导数都连续,则称 \(f\in\scr{C}^1\)
  • 对于 \(k\ge 1\),如果 \(f\) 在定义域中每个点处都存在一阶偏导数,并且所有一阶偏导数都是 \(\scr{C}^{k-1}\) 型的,则称 \(f\in\scr C^k\)

对于多元函数而言,\(\scr C^1\) 是最强的条件,其次是可微,再其次是连续和偏导数存在(连续和偏导数存在这两者没有必然联系)

多元函数的泰勒展开与极值

泰勒展开

对于函数 \(f\in\scr{C}^r\) 和其定义域中某点 \(\bold{x_0}\),和某个方向向量 \(\bold{v}\),如果记 \(g(t)=f(\bold{x_0}+t\bold{v})\),那么根据一元函数的泰勒展开可知

\[g(t)=\sum\limits_{i=0}^rg^{(i)}(0)\dfrac{t^i}{i!}+o(t^r),t\to 0 \]

而因为

\[g'(t)=\mathrm df(\bold{x_0}+t\bold{v})\bold{v}=\sum\limits_{k=1}^mf_{k}(\bold{x_0}+t\bold{v})\bold{v}^k \]

\[g''(t)=\sum\limits_{k=1}^m\mathrm df_k(\bold{x_0}+t\bold{v})\bold{v}\bold{v}^k=\sum\limits_{i=1}^m\sum\limits_{j=1}^mf_{ij}(\bold{x_0}+t\bold{v})\bold{v}^j\bold{v^k} \]

归纳可得

\[g^{(k)}(t)=\sum\limits_{j_1,j_2,\cdots,j_k\in\{1,2,\cdots,m\}}f_{j_1,j_2,\cdots,j_m}(\bold{x_0})\bold{v}^{j_1}\bold{v}^{j_2}\cdots\bold{v}^{j_m} \]

于是

\[f(\bold{x_0}+t\bold{v})=\sum\limits_{k=0}^r\sum\limits_{j_1,j_2,\cdots,j_k\in\{1,2,\cdots,m\}}f_{j_1,j_2,\cdots,j_m}(\bold{x_0})\bold{v}^{j_1}\bold{v}^{j_2}\cdots\bold{v}^{j_m}\dfrac{t^k}{k!}+o(t^r),t\to 0 \]

上式也可以写作

\[f(\bold{x_0}+\bold{v})=\sum\limits_{k=0}^r\sum\limits_{j_1,j_2,\cdots,j_k\in\{1,2,\cdots,m\}}f_{j_1,j_2,\cdots,j_m}(\bold{x_0})\bold{v}^{j_1}\bold{v}^{j_2}\cdots\bold{v}^{j_m}\dfrac{1}{k!}+o(\lVert\bold{v}\lVert^r),\lVert\bold{v}\rVert\to 0 \]


因为高阶连续可微函数的高阶偏导数与求导顺序无关,所以

\[f(\bold{x_0}+\bold{v})=\sum\limits_{k=0}^r\sum\limits_{\sum\alpha_i=k}\dfrac{f^{(\alpha_1,\alpha_2,\cdots,\alpha_m)}(\bold{x_0})(\bold{v}^1)^{\alpha_1}(\bold{v}^2)^{\alpha_2}\cdots(\bold{v}^m)^{\alpha_m}}{\alpha_1!\alpha_2!\cdots\alpha_m!}+o(\lVert\bold{v}\lVert^r),\lVert\bold{v}\rVert\to 0 \]

其中 \(f^{(\alpha_1,\alpha_2,\cdots,\alpha_m)}(\bold{x_0})=\dfrac{\partial^kf(\bold{x_0})}{\partial(x_1)^{\alpha_1}\partial(x_2)^{\alpha_2}\cdots\partial(x_m)^{\alpha_m}}\)

上式便是 \(f\)\(\bold{x_0}\) 处的 \(r\) 阶 Peano 余项的泰勒展开式。

类似地也可以得到 \(f\)\(\bold{x_0}\) 处的 \(r\) 阶 Lagrange 余项的泰勒展开式。

\[f(\bold{x_0}+\bold{v})=\sum\limits_{k=0}^{r}\sum\limits_{\sum\alpha_i=k}\dfrac{f^{(\alpha_1,\alpha_2,\cdots,\alpha_m)}(\bold{x_0})(\bold{v}^1)^{\alpha_1}(\bold{v}^2)^{\alpha_2}\cdots(\bold{v}^m)^{\alpha_m}}{\alpha_1!\alpha_2!\cdots\alpha_m!}+\sum\limits_{\sum\alpha_i=r+1}\dfrac{f^{(\alpha_1,\alpha_2,\cdots,\alpha_m)}(\bold{x_0}+\xi\bold{v})(\bold{v}^1)^{\alpha_1}(\bold{v}^2)^{\alpha_2}\cdots(\bold{v}^m)^{\alpha_m}}{\alpha_1!\alpha_2!\cdots\alpha_m!} \]

其中 \(0<\xi<1\)

多元函数的极值

考虑可微多元函数 \(f\) 定义域中某一点 \(\bold{x_0}\),如果 \(\mathrm df(\bold{x_0})\ne 0\),那么存在 \(\bold{v}\) 使得 \(\mathrm df(\bold{x_0})\bold{v}>0\),这样对于足够小的正数 \(t\),有 \(f(\bold{x_0}-t\bold{v})<f(\bold{x_0})<f(\bold{x_0}+t\bold{v})\)

也就是说,如果 \(\bold{x_0}\)\(f\) 的极值点,那么 \(\mathrm df(\bold{x_0})=0\)

但是这个条件不是充分必要的,反例:鞍点。要想知道一个微分为 \(0\) 的点是否真正为极值点,需要对二阶微分加以分析。

考虑 \(m\times m\) 的矩阵 \(H_f(\bold{x_0})\),其第 \(i\) 行第 \(j\) 列的值为 \(f_{i,j}(\bold{x_0})\),有结论:

  • 如果 \(H_f(\bold{x_0})\) 正定,那么 \(\bold{x_0}\)\(f\) 的极小值点。
  • 如果 \(H_f(\bold{x_0})\) 负定,那么 \(\bold{x_0}\)\(f\) 的极大值点。

其中,一个 \(n\times n\) 的矩阵 \(A\) 被称为正定的,当且仅当 \(\forall\bold{x}=(x_1,x_2,\cdots,x_n)\ne\bold{0}\),都有 \(\bold{x} A\bold{x}^T>0\);一个 \(n\times n\) 的矩阵 \(A\) 被称为负定的,当且仅当 \(\forall\bold{x}=(x_1,x_2,\cdots,x_n)\ne\bold{0}\),都有 \(\bold{x} A\bold{x}^T<0\)

特别地对于二阶矩阵 \(H=\begin{bmatrix}\alpha&\beta\\\beta&\gamma\end{bmatrix}\),有:

  • \(H\) 正定:\(\alpha>0,\alpha\gamma>\beta^2\)
  • \(H\) 负定:\(\alpha<0,\alpha\gamma>\beta^2\)
  • \(H\) 退化:\(\alpha\gamma=\beta^2\)
  • \(H\) 非退化、非正定、非负定:\(\alpha\gamma<\beta^2\)

隐函数定理与逆映射定理

隐函数定理

对于 \(\scr C^r\) 的映射 \(f:\mathbb{R}^m\times\mathbb{R}^n\to\mathbb{R}^n\),且已知 \(f(\bold{x_0},\bold{y_0})=0\)\(Jf(\bold{x_0},\bold{y_0})\) 可逆,则存在 \(\bold{x_0}\) 的某个邻域 \(U\)\(\bold{y_0}\) 的某个邻域 \(V\),以及 \(\scr C^r\) 映射 \(g:U\to V\) 满足 \(\forall\bold{x}\in U,\bold{y}\in V\)\(f(\bold{x},\bold{y})=0\Leftrightarrow \bold{x}=g(\bold{y})\)。并且

\[\mathrm Dg(\bold{x})=-(\mathrm DF_y(\bold{x},g(\bold{x})))^{-1}\mathrm DF_x(\bold{x},g(\bold{x})) \]

换句话说,在 \(\bold{x_0}\) 的邻域内,方程的解存在且唯一,且解函数的连续性与原函数连续性相当。

逆映射定理

对于映射 \(H\in\scr C^r\),若 \(\bold{y_0}=H(\bold{x_0})\),且 \(H_{\bold{x}}(\bold{x_0})\) 可逆,那么根据隐函数定理,存在 \((\bold{x_0},\bold{y_0})\) 的一个邻域 \(W\)\(\scr C^r\) 的映射 \(g\) 满足 \(\forall(g(\bold{y}),\bold{y})\in W\)\(H(g(\bold{y}))=\bold{y}\)。换句话说,\(g\)\(H\)\((\bold{x_0},\bold{y_0})\) 附近的局部逆映射,

整体微分同胚定理

对于开集 \(U\subset\mathbb{R}^n\)\(\scr C^r\) 的映射 \(F:U\to\mathbb{R}^n\),那么以下两个陈述等价:

  • \(V=F(U)\)\(\mathbb{R}^n\) 中的开集,并且存在 \(\scr C^r\) 的逆映射 \(F^{-1}:V\to U\)(记作 \(F\)\(\scr C^r\) 微分同胚)
  • \(F\) 是单射,且 \(\forall\bold{x}\in U\)\(\mathrm DF(\bold{x})\) 可逆。

曲线与曲面

曲线的定义

对于空间 \(\mathbb{R}^m\),定义 \(\mathbb{R}^m\) 中的一个 \(\scr C^r\) 曲线是一个 \(\scr C^r\) 映射 \(\bold{x}:(\alpha,\beta)\to\mathbb{R}^m\),其中 \(\bold{x}'(t)\) 为该曲线在 \(\bold{x}(t)\) 处的速度向量。定义一个曲线是正则的,当且仅当 \(\bold{x}'(t)\ne 0\)

曲面的定义

对于 \(\Sigma\subset\mathbb{R}^n\),称 \(\Sigma\) 是一个 \(m\)\(\scr C^r\) 曲面,当且仅当 \(\forall P_0\in\Sigma\),都存在 \(P_0\) 的一个开邻域 \(U\)\(V\subset\mathbb{R}^m\)\(\scr C^r\) 映射 \(g:V\to\mathbb{R}^{n-m}\) 和一个排列 \(\sigma\) 满足 \(\forall (x^1,x^2,\cdots,x^n)\in\Sigma\cap U\) 使得 \((x^{\sigma(m+1)},x^{\sigma(m+2)},\cdots,x^{\sigma(n)})=g(x^{\sigma(1)},x^{\sigma(2)},\cdots,x^{\sigma(m)})\)

特别地,当 \(m=1\) 时,\(\Sigma\)\(\mathbb{R}^n\) 中的一个曲线。

定义一个曲面是正则的,当且仅当它的 Jacobi 矩阵列满秩。


对于映射 \(F:U\to \mathbb{R},U\subset\mathbb{R}^n,F\in\scr C^r\),如果 \(a\in\mathbb{R}\)\(F\) 的一个正则值(即 \(F^{-1}\) 非空且不含 \(F\) 的临界点),则水平集 \(F^{-1}(a)\) 是一个 \(n-1\) 维的 \(\scr C^r\) 曲面。

证明:根据正则值的定义,\(\forall P_0=(x_0^1,x_0^2,\cdots,x_0^n)\in F^{-1}(a)\)\(F\) 至少有一个偏导数 \(F_k(P_0)\ne 0\)。于是由隐函数定理,在 \(P_0\) 附近存在 \(\scr C^r\) 函数 \(g:V\to\mathbb{R}\) 使得 \(F(\bold{x})=a\),当且仅当 \(x^k=g(\hat{\bold{x}})\),其中 \(\hat{\bold{x}}\)\(\bold{x}\) 去掉第 \(k\) 维后的结果。

类似地可以推广得到:对于映射 \(F:U\to\mathbb{R}^{n-m},U\subset\mathbb{R}^n,F\in\scr C^r\),如果 \(a\in\mathbb{R}^{n-m}\)\(F\) 的一个正则值,即 \(F^{-1}(a)\) 非空,且 \(\forall\bold{x}\in F^{-1}(a)\)\(\mathrm DF^{-1}(\bold{x})\) 满秩。则 \(F^{-1}(a)\) 是一个 \(m\) 维的 \(\scr C^r\) 曲面。

曲面的切向量、切空间、切平面

对于 \(\mathbb{R}^n\) 中的 \(\scr C^r\) 曲面和 \(\mathbb{R}^n\) 中一条 \(\scr C^1\) 路径 \(\bold{x}(t)\),满足 \(\forall t,\bold{x}(t)\in\Sigma\),如果 \(\bold{x}(0)=P_0\),则 \(\bold{x}'(0)\)\(P_0\) 处一个切向量。所有切向量组成的集合称为 \(P_0\) 处的切空间,记作 \(T_{P_0}(\Sigma)\)\(P_0+T_{P_0}(\Sigma)\) 称为 \(P_0\) 处的切平面。切平面是切空间沿着 \(P_0\) 这个向量平移以后的结果。

对于曲线

\[\begin{cases} x^1=x^1(t^1,t^2,\cdots,t^m)\\ x^2=x^2(t^1,t^2,\cdots,t^m)\\ \vdots\\ x^m=x^m(t^1,t^2,\cdots,t^m) \end{cases} \]

\(\bold{v_1}=\dfrac{\partial\bold{x}}{\partial t_1},\bold{v_2}=\dfrac{\partial\bold{x}}{\partial t_2},\cdots,\bold{v_m}=\dfrac{\partial\bold{x}}{\partial t_m}\)\(m\) 个线性无关的向量,一个点处的切平面则是由这个点处的 \(\bold{v_1},\bold{v_2},\cdots,\bold{v_m}\) 所张成的空间,即其 Jacobi 矩阵 \(J\bold{x}(\bold{t})\) 的列空间。

切平面上的点在 \(\mathbb{R}^n\) 坐标系下,\(\bold{x}=\bold{x}(\bold{t_0})+\sum\limits_{k=1}^m\dfrac{\partial\bold{x}}{\partial t^k}(\bold{t_0})\xi^k\),其中 \(\xi^k\in\mathbb{R}\)

\(\mathbb{R}^3\) 中的 \(\mathbb{R}^2\) 曲面

隐函数确定的平面

隐函数确定的平面指用方程 \(F(x,y,z)=0\) 确定的二维曲面,其中 \(F_x,F_y,F_z\) 中至少有一个非零。

对于曲面上任意一条光滑曲线 \(\bold{x}(t)=(x(t),y(t),z(t))\),因为 \(F(x(t),y(t),z(t))=0\),两边同时对 \(t\) 求导可得 \(\nabla F·\bold{x}'(t)=0\)。换句话说,曲面在 \(\bold{x_0}\) 处的切向量总与 \(F\)\(\bold{x_0}\) 处的梯度向量(即法向量)正交。而对于任意一个与在 \(\bold{x_0}\) 处与 \(F\)\(\bold{x_0}\) 处的梯度向量正交的向量 \(\bold{v}\),都存在光滑曲线 \(\bold{x}(t)\) 使得 \(F(\bold{x(t)})=0\),并且 \(\bold{x}(0)=\bold{x_0},\bold{x}'(0)=\bold{v}\)。也就是说对于曲面上一点 \(\bold{x_0}\),曲面的切平面可以表示为

\[\nabla F(\bold{x_0})(\bold{x}-\bold{x_0})=0 \]

因为 \(\nabla F(\bold{x_0})\) 是好求的,所以通过这种方式可以求得曲面上一点的切平面。

类似地,对于用隐函数 \(\begin{cases}F_1(x,y,z)=0\\F_2(x,y,z)=0\end{cases}\) 定义的曲线,其在 \((x_0,y_0,z_0)\) 处的切方向就是 \(\nabla f_1(x_0,y_0,z_0)\)\(\nabla f_2(x_0,y_0,z_0)\) 的叉积所对应的方向。

参数方程确定的曲面

参数方程确定的曲面指用参数方程 \(\bold{x}(u,v):=(x(u,v),y(u,v),z(u,v))\) 确定的二维曲面,其中 \((u,v)\in D\subseteq\mathbb{R}^2\),并且 \(\dfrac{\partial(x,y)}{\partial(u,v)},\dfrac{\partial(y,z)}{\partial(u,v)},\dfrac{\partial(x,z)}{\partial(u,v)}\) 中至少有一个可逆。

对于曲面上任意一条光滑曲线 \(\bold{y}(t)=\bold{x}(u(t),v(t))\),两边同时对 \(t\) 求导可得 \(\bold{y}'(t)=\dfrac{\partial\bold{x}}{\partial u}|_{\bold{x=y(t)}}u'(t)+\dfrac{\partial\bold{x}}{\partial v}|_{\bold{x=y(0)}}v'(t)\),因为 \(u'(t)\)\(v'(t)\) 可以取到任意值,所以曲面在 \(\bold{x_0}\) 处的切空间是由该点处 \(\bold{x}\)\(u,v\) 求偏导数所得的向量的张成空间,即向量 \(\dfrac{\partial\bold{x}}{\partial u}\)\(\dfrac{\partial\bold{x}}{\partial v}\) 所张成的空间。类似地也可以得到,曲线在该点处的法向量就是向量 \(\dfrac{\partial\bold{x}}{\partial u}\)\(\dfrac{\partial\bold{x}}{\partial v}\) 的叉积。

条件极值问题

约束条件极值问题形如,给定 \(k\) 个方程组 \(\begin{cases}g_1(\bold{x})=0\\g_2(\bold{x})=0\\\cdots\\g_k(\bold{x})=0\end{cases}\),要求满足这 \(k\) 个方程组的 \(f(\bold{x})\) 的最值。

这种问题可以使用 Lagrange 乘子法求解,即考虑函数 \(L(\bold{x},\lambda_1,\lambda_2,\cdots,\lambda_k)=f(\bold{x})-\lambda_1g_1(\bold{x})-\lambda_2g_2(\bold{x})-\cdots-\lambda_kg_k(\bold{x})\),如果 \((\bold{x}^*,\lambda_1^*,\lambda_2^*,\cdots,\lambda_k^*)\) 为这个函数的极值点,那么需要满足:

\[\begin{cases} \nabla f(\bold{x}^*)-\lambda_1^*\nabla g_1(\bold{x}^*)-\lambda_2^*\nabla g_2(\bold{x}^*)-\cdots-\lambda_k^*\nabla g_k(\bold{x}^*)=0\\ g_i(\bold{x}^*)=0 \end{cases} \]

第一条限制保证了 \(f\) 的梯度向量与 \(g\)\(\bold{x}^*\) 处的切平面正交,第二条限制保证了 \(\bold{x}^*\) 符合 \(g\)\(k\) 条限制,因此存在 \(\lambda_1,\lambda_2,\cdots,\lambda_k\) 使得 \((\bold{x}^*,\lambda_1,\lambda_2,\cdots,\lambda_k)\)\(L\) 的极值点是 \(\bold{x}^*\)\(f\) 的条件极值的必要条件,那如何检验 \(\bold{x}\) 是否真的是 \(f\) 的极值点呢,有三种方法:

  • 如果符合 \(g\) 的限制的 \(\bold{x}\) 组成有界闭集,并且 \(f\) 为连续函数,那么最值一定是极值,直接算出所有候选点集的 \(f\) 的最小值就是全局最小值,最大值也同理。
  • 如果符合 \(g\) 的限制的 \(\bold{x}\) 不是有界闭集,\(f\) 为连续函数,并且 \(\bold{x}\) 趋近于 \(g\) 的边界时值为 \(\infty\),那么最小值点一定是极值点;\(\bold{x}\) 趋近于 \(g\) 的边界时值为 \(-\infty\) 的情况也同理。
  • 其他情况:找出切空间以后计算二阶导数从而判断 Hesse 矩阵是正定还是负定还是非正定非负定,即考虑所有符合要求的切向量 \((\xi^1,\xi^2,\cdots,\xi^k)\),判断 \((\xi^1,\xi^2,\cdots,\xi^k)H_f(\bold{x})(\xi^1,\xi^2,\cdots,\xi^k)^T\) 的符号:
    • 恒大于 \(0\):极小值。
    • 恒小于 \(0\):极大值。
    • 不固定:啥都不是。

广义含参积分

对于二元函数 \(f(t,\bold{x})\),设 \(g(\bold{x})=\int_a^{\omega}f(t,\bold{x})\),其中积分在 \(\omega\) 处为瑕点。在广义含参积分中,除了探讨积分 \(g(\bold{x})\) 是否收敛之外,我们往往还会探讨 \(g\) 的整体性质,比如 \(g\) 的连续性、可微性、可积性等。

一致收敛

\(g(\bold{x})=\int_a^{\omega}f(t,\bold{x})\) 关于 \(\bold{x}\)\(A\) 上一致收敛,当且仅当 \(\forall\epsilon>0\),存在 \(\omega\) 的去心邻域 \(U_{\epsilon}\) 使得 \(\forall x\in A\),对任何 \(b\in U_{\epsilon}\),都有

\[|\int_{a}^bf(t,\bold{x})\mathrm dt-g(\bold{x})|<\epsilon \]

换句话说,就是对于所有参数 \(\bold{x}\),广义积分 \(f(t,\bold{x})\) 收敛的速度相同。

广义积分的连续性

\(A\) 是开集或闭集,\(f(t,\bold{x})\)\([a,\omega)\times A\) 上连续,并且 \(g(\bold{x})\)\(A\) 上一致收敛,那么 \(g(\bold{x})\) 连续。

证明:

因为

\[\begin{aligned} |g(\bold{x})-g(\bold{x_0})|&=|\int_a^{\omega}f(t,\bold{x})\mathrm dt-\int_a^{\omega}f(t,\bold{x_0})|\\ &=|\int_b^{\omega}f(t,\bold{x})\mathrm dt+\int_a^b(f(t,\bold{x})-f(t,\bold{x_0}))\mathrm dt-\int_b^{\omega}f(t,\bold{x})\mathrm dt|\\ &\le|\int_b^{\omega}f(t,\bold{x})\mathrm dt|+\int_a^b|f(t,\bold{x})-f(t,\bold{x_0})|\mathrm dt+|\int_b^{\omega}f(t,\bold{x})\mathrm dt| \end{aligned} \]

因为 \(g\)\(A\) 上一致收敛,所以存在 \(b(\epsilon\)) 使得 \(\forall\bold{x}\in A\)\(|\int_b^{\omega}f(t,\bold{x})\mathrm dt|<\dfrac{\epsilon}{3}\)。因为 \(f\) 在有界闭集上连续,从而一致连续,故存在 \(\delta(\epsilon)\) 使得 \(\forall\lVert\bold{x}-\bold{x_0}\rVert<\delta(\epsilon)\)\(a\le t\le b\),都有 \(|f(t,\bold{x})-f(t,\bold{x_0})|<\dfrac{\epsilon}{3(b-a)}\),这样 \(|g(\bold{x})-g(\bold{x_0})|<\epsilon\)

广义积分的可积性

\(f(t,s)\) 关于 \(t,s\)\([a,\omega)\times[\alpha,\beta]\) 连续,并且 \(g(s)\) 关于 \(t\)\([\alpha,\beta]\) 上一致收敛,那么

\[\int_{\alpha}^{\beta}\int_a^{\omega}f(t,s)\mathrm dt\mathrm ds=\int_a^{\omega}\int_{\alpha}^{\beta}f(t,s)\mathrm ds\mathrm dt \]

证明:\(\forall\epsilon>0\),因为 \(g(s)\) 关于 \(t\)\([\alpha,\beta]\) 上一致收敛,考虑找出 \(b\) 使得 \(\forall s\in[\alpha,\beta]\),都有 \(|\int_b^{\omega}f(t,s)\mathrm dt|<\dfrac{\epsilon}{\beta-\alpha}\),这样

\[\begin{aligned} &|\int_a^b\int_{\alpha}^{\beta}f(t,s)\mathrm ds\mathrm dt-\int_{\alpha}^{\beta}\int_a^{\omega}f(t,s)\mathrm dt\mathrm ds|\\ =&\int_{\alpha}^{\beta}|\int_b^{\omega}f(t,s)\mathrm dt|\mathrm ds\\ <&\epsilon \end{aligned} \]

因此 \(\int_{\alpha}^{\beta}\int_a^{\omega}f(t,s)\mathrm dt\mathrm ds=\int_a^{\omega}\int_{\alpha}^{\beta}f(t,s)\mathrm ds\mathrm dt\)

该定理可以进一步推广:若 \(f(t,s)\) 关于 \(t,s\)\([a,\omega_1)\times[\alpha,\omega_2)\) 连续,并且 \(\int_a^{\omega_1}f(t,s)\mathrm dt\) 关于 \(s\)\([\alpha,\omega_2)\) 上一致收敛,\(\int_a^{\omega_2}f(t,s)\mathrm ds\) 关于 \(t\)\([a,\omega_1)\) 上一致收敛,并且 \(\int_a^{\omega_1}\int_{\alpha}^{\omega_2}|f(t,s)|\mathrm dt\mathrm ds\)\(\int_{\alpha}^{\omega_2}\int_a^{\omega_1}|f(t,s)|\mathrm ds\mathrm dt\) 至少有一者收敛,则 \(\int_a^{\omega_1}\int_{\alpha}^{\omega_2}f(t,s)\mathrm dt\mathrm ds\)\(\int_{\alpha}^{\omega_2}\int_a^{\omega_1}f(t,s)\mathrm ds\mathrm dt\) 均收敛并且值相等。

广义积分的可微性

对广义积分 \(g(\bold{x})=\int_a^{\omega}f(t,\bold{x})\mathrm dt\) 而言,\(g\) 的偏导数 \(\dfrac{\partial}{\partial x_k}g(\bold{x})\) 存在,当且仅当:

  • 存在 \(\forall\bold{x}\in A\)\(g(\bold{x})\) 收敛。
  • \(\dfrac{\partial}{\partial x_k}f(t,\bold{x})\) 关于 \((t,\bold{x})\) 二元连续,并且 \(\int_a^{\omega}\dfrac{\partial}{\partial x_k}f(t,\bold{x})\) 关于 \(\bold{x_0}\) 一致收敛。

此时 \(\dfrac{\partial}{\partial x_k}g(\bold{x})=\int_a^{\omega}\dfrac{\partial}{\partial x_k}f(t,\bold{x})\),即偏导运算与积分运算可以交换。

判定广义积分一致收敛的办法

如果使用一致收敛的定义去判断广义积分是否一致收敛,通常需要先求出 \(g(\bold{x})\) 的值以后才能判断,因此有以下三种更普遍的方法:

1. 一致 Cauchy

如果 \(\forall\epsilon>0,\bold{x}\in A\),存在 \(\omega\) 的去心邻域 \(U_{\epsilon}\) 使得 \(\forall b_1,b_2\in U_{\epsilon}\),都有

\[|\int_{b_1}^{b_2}f(t,\bold{x})\mathrm dt|<\epsilon \]

那么 \(f(t,\bold{x})\) 关于 \(\bold{x}\)\(A\) 上一致收敛。

2. Weierstress 强函数

如果存在 \(g(t)\) 满足 \(\int_a^{\omega}|g(t)|\mathrm dt\) 收敛且存在 \(\omega\) 的去心邻域 \(U\) 使得 \(\forall\bold{x}\in A,t\in U\),都有 \(|f(t,\bold{x})|\le|g(t)|\),则 \(\int_a^{\omega}|f(t,\bold{x})|\mathrm dt\) 关于 \(\bold{x}\)\(A\) 上一致收敛。

Weierstress 同样使用于一致绝对收敛。

3. 乘积函数判别法(Dirichlet-Abel)

对于广义积分 \(\int_a^{\omega}f(t,\bold{x})g(t,\bold{x})\mathrm dt\) 型的积分,当满足以下两个条件之一时,积分关于 \(\bold{x}\) 一致收敛:

  • Dirichlet 判别法:\(\int_a^{b}f(t,\bold{x})\mathrm dt\) 对所有 \(b\in[a,\omega)\) 关于 \(\bold{x}\) 一致有界,并且 \(\lim\limits_{t\to \omega}g(t,\bold{x})=0\)\(\bold{x}\) 一致成立。
  • Abel 判别法:\(\int_a^{\omega}f(t,\bold{x})\mathrm dt\)\(\bold{x}\) 一致收敛,并且 \(g(t,\bold{x})\) 有界。

那么广义积分 \(\int_a^{\omega}f(t,\bold{x})g(t,\bold{x})\mathrm dt\) 关于 \(\bold{x}\) 一致收敛。

posted @ 2024-02-28 22:21  tzc_wk  阅读(340)  评论(13编辑  收藏  举报